The question stem reads: Which one of the following, if true, most weakens the argument? This is a weaken question.

The author describes two groups of people who have chronic trouble falling asleep. One group relies only on sleeping pills, while another relies only on behavior modification to treat their maladies. The group that relies on behavior modification falls asleep more quickly than the group that relies on sleeping pills. The author concludes from this that behavior modification is more effective than sleeping pills in helping people fall asleep.

The author has made the causal claim: the author argues that behavior modification is causing that group to fall asleep faster than the group who uses sleeping pills. In the curriculum, we covered how the tool science uses to identify causes is the Ideal Experiment. One of the requirements of the Ideal Experiment is that we randomly assign the samples into experimental and control groups. Random assignment of the groups helps isolate the hypothesized cause by controlling for other causes.

The author has provided the hypothesis that behavior modification is causing that group to fall asleep faster than the sleeping pill group. If this were an ideal experiment, we would expect to see a large sample size of people who had chronic trouble falling asleep assigned into a behavior modification group, a sleeping pill group, and a control group. However, what we find in the stimulus is not an experiment with randomly assigned groups. Instead, we find an observation that people who use behavior modification tend to fall asleep faster than those who use sleeping pills. By failing to assign groups randomly, we fail to isolate for other causes, such as self-selection. What does self-selection mean in this case? Let me ask you, “Why might it be that some people use sleeping pills (a medical intervention) and others use behavior modification (a change in habits)?” If you think that people who use sleeping pills might have a more severe case of insomnia than those who use behavior modification, you are spot on. The individuals who have a less severe case of insomnia might have simply self-selected into using behavior modification. The fact they fall asleep quicker might not be caused by their treatment. Instead, they simply have a less severe form of the disease.

Our job is to weaken the argument. So a good answer choice will illustrate the problem we just identified.

Answer Choice (A) is irrelevant to the argument. The argument is about the time it takes to fall asleep, while (A) speaks to the amount of sleep different groups get over the course of a night.

Answer Choice (B) fails to weaken the argument by comparing the behavioral modification group to a newly introduced group: one that has no trouble falling asleep. We can rule this answer choice out because the argument is solely concerned with individuals who need help falling asleep. The conclusion is a comparative claim between behavior modification and sleeping pills, so the comparison between behavior modification and people who do not have trouble falling asleep is irrelevant.

Answer Choice (C) fails to weaken because the argument is already explicitly concerned with the group that uses behavioral modification and does not use sleeping pills. Whether or not people have or have not used sleeping pills in the past is arbitrary.

Correct Answer Choice (D) is exactly what we prephrased. (D) States that the people who are most likely to use sleeping pills are those who have the most trouble falling asleep. So those who use sleeping pills could be falling asleep slower because they initially had much more trouble falling asleep than those who decided to use behavioral modification.

Answer Choice (E) is incorrect. While it does expose a self-selection bias (Those who choose the behavior modification prefer it to medication), it is unclear exactly how that would affect the time it takes to go to sleep. (D) provides a much clearer reason as to why the pill group takes longer.

Cookie Cutters
39.2.05
25.4.24


13 comments

The Question Stem reads: The lawyer's conclusion follows logically if which one of the following is assumed? This is a Sufficient Assumption question.

The lawyer begins by describing how this witness was present at the restaurant when the lawyer's famous client was assaulted. However, the witness claims to recognize the assailant but not the famous client. The lawyer concludes the witness's testimony should be excluded. We can break down the argument to read:

P1: Witness claims to recognize the assailant

P2: Witness claims not to recognize the victim (Famous client)

________________

C: Exclude witness testimony

In the CC, we discussed that elements of the conclusion must be in the premises. Nowhere in the premises do we see a claim about what kind of testimony should not be included. We need a conditional that brings us to "exclude testimony," so let's make that our necessary condition: (__) -> Exclude testimony.

Now it will be hard to anticipate what sufficient condition the AC will use. They could use P1 or P2 or some combination of both. When we screen these answer choices, the first order of business will be to make sure that the necessary condition is: "exclude testimony" Then we will check the sufficient condition to ensure it gets triggered by the information in the stimulus. Let's go.

Answer Choice (A) has the necessary condition "then the witness's testimony should be included." Without looking at the rest of the (A), we can eliminate it because it takes us to "include" when we want to go to "exclude." If you picked (A), you likely assumed that "claims recognize both parties -> include" implied that "/(claims recognize both parties) -> exclude," which is a logical fallacy. Remember: a->b does not imply /a->/b.

Answer Choice (B) is arbitrary. Why would the fact that other witnesses can identify the client mean we should exclude the witness from the stimulus? Nothing. Eliminate and move on.

Answer Choice (C) is also arbitrary because it does not bring us to a conclusion "exclude testimony." As a side note, whether or not we can know if the witness actually recognized the assailant is irrelevant. Notice how the premises only take into account who the victim claims to recognize. The lawyer's argument is going to rely on the witness's claims, not what actually is the case.

Correct Answer Choice (D) gets us to where we need to go. If we take the contrapositive of (D), we get: "/(claims to recognize both parties in assault) -> exclude." The necessary condition is exactly what we discussed. The sufficient condition is great. The witness did not claim to recognize both parties in the assault. The witness claims to recognize only the assailant. So the conditional triggers and delivers us to the conclusion that the witness's testimony should be excluded.

Answer Choice (E) is a popular wrong answer. If you picked (E), you likely inferred that the witness was lying about not recognizing the famous victim. First, that is not an inference you can make. Just because it is unlikely that someone wouldn't recognize the famous client, that does not mean it is impossible that someone would fail to recognize the client. Second, even if we could infer that the witness was lying, it wouldn't help us. What does lying have to do with excluding testimony? If you answered that "liars testimony should be excluded," you've proved that (E) by itself is insufficient to draw the lawyer's argument.


8 comments

The question stem reads: The biologist’s argument is most vulnerable to criticism on which one of the following grounds? This is a Flaw question.

The biologist begins by describing how many paleontologists suggest that the difficulties of the ice age were a cause of the evolution of the human brain. The biologist concludes those palentologists are wrong. In other words, the ice age was not responsible for the evolution of the human brain. As evidence, the biologist cites that many animal species survived the ice age with no evolutionary changes to their brain.

The biologist has hypothesized that the ice age was not repsonsbile for the evolution of the human brain. If we were to construct an ideal experiment to test this hypothesis, what kind of subjects would you want to use? You would want to use human brains! However, the biologist instead uses animals brains as evidence. The question is, “Are humans and animals” the same? Maybe. Maybe not. The biologist’s argument relies on the assumption that humans and animals would respond to the evolutionary pressures of the ice age in the same way. So let’s look for an answer choice that distinguishes how animal brains and human brains would respond to the ice age.

Answer Choice (A) is incorrect. The biologist does not suggest that the ice age was sufficient or necessary to produce brain evolution in humans or animals.

Correct Answer Choice (B) draws the distinction between humans and animals that we are looking for. The biologist fails to consider the possibility that the ice age could have produced the evolution of the brain in humans without producing the evolution of the brain in other species.

Answer Choice (C) is incorrect. The argument is not about whether the ice age was necessary for producing changes in the brains of humans or animals. The argument is about whether the ice age was sufficient to bring about changes in the brains of humans and animals.

Answer Choice (D) is incorrect. The biologist never presumes that humans had a more difficult time during the ice age than animals.

Answer Choice (E) is incorrect. The biologist does not presume that the ice age was causally responsible for the evolution of human brains. He concludes that the ice age was not casusualy responsible for the evolution of human brains.


15 comments

We’ve got a Main Conclusion question here, which we can ID from the question stem: Which one of the following most accurately expresses the conclusion drawn in the botanist’s argument?

Our stimulus starts with a classic set up for some context: “it has long been believed that...” This is essentially another version of the “other people’s opinion” construct that so often introduces context. Who exactly has been believing this idea? Well people of course! Namely, other people. See? Just another way of introducing OPO–a classic set up for some context.

What follows is a description of a belief that we can start to anticipate will likely be refuted or partially challenged by our argument. Whenever we start with a consensus view attributed implicitly or explicitly to other people, we usually move into an argument that takes issue with or qualifies that view in some way. In this case we’re told that people have long believed that we should keep poinsettias out of homes with kids or pets.

We then get some more context: “this belief” (a referential phrase referring to the belief about poinsettias being kept out of homes) has been espoused in parenting books.

Now we get what definitely feels like our conclusion: “it” (another referential phrase for the idea that we should keep poinsettias out of the home) is mistaken. But wait, doesn’t a conclusion need to be supported by a premise? How do we know this is our conclusion? Great questions! You all deserve gold stars!

We follow up this idea with a sentence that provides that direct support that we so desperately need. The third sentence gives us an explanation as to why this belief is mistaken: research demonstrates that poinsettias actually pose no threat to kids or pets. This is a premise giving us rationale for our conclusion, which is: “it [the belief that poinsettias should be kept out of the home] is mistaken.”

Answer Choice (A) This is a gross misreading of the stimulus. Our conclusion states that the idea that we need to keep poinsettias out of the home “is mistaken.” This is not the same thing as advocating for the entirely different idea of putting poinsettias in the home. Taking issue with an idea does not equate to advocating for a totally new position that moves in the opposite direction. On top of that we have this completely unsupported notion that child rearing books should do something differently which is way outside the scope of our conclusion.

Answer Choice (B) This is a reasonable reading of our premise. If poinsettias pose no risk to kids or pets then we can reasonably conclude that poinsettias are not dangerously poisonous. Unfortunately, it doesn’t describe our conclusion, so is therefore, incorrect.

Answer Choice (C) This is consistent with our context: if child-rearing books have commonly listed poinsettias as dangerous it’s fairly reasonable to say that they view these plants as dangerous. But, alas, it does not at all describe our conclusion.

Correct Answer Choice (D) This is a perfect description of our main conclusion. It even fleshes out what the referential word “it” means (i.e. the belief that households with children or pets should not have poinsettias).

Answer Choice (E) Again, we have another answer choice that is an accurate description of our premise. It even uses the same wording! But it is out of the scope of our conclusion which narrowly focuses on a specific belief (keeping poinsettias out of the house) being “mistaken”.


4 comments

We’ve got a MSS question which we can tell from the question stem: If the statements above are true, which one of the following is the most strongly supported on the basis of them?

The stimulus starts with what looks like a sentence of contextual information which introduces us to a category: common threats to life. The embedded phrase between the commas gives us examples of common threats to life: automobile and industrial accidents. We’re told that only unusual instances of common threats like these receive coverage from the news media.

We then have a shift into what looks like our potential argument (remember that MSS requires us to find a conclusion that completes said argument). This shift is indicated by the “however” that’s wedged into the middle of the next sentence. We then get a premise that introduces us to another category: rare threats. We also get an example of rare threats: product tampering. We are told that instances of rare threats are seen as news and universally reported by reporters in featured stories.

Ok so we know that the media always reports instances of rare threats, but only reports unusual instances of common threats. We then get some information about how the media impacts the population at large: people tend to estimate the risk of threats based on how often those threats come to their attention.

What’s a common way things come to our attention? Well, the media, right? Do we see a potential connection here?

Media covers rare threats instead of common threats. People see coverage of rare threats but not common threats. People perceive greater risk from rare threats because they estimate risk based on how often something comes to their attention.

Seems like a good synthesis of the information we’ve been given! Now let’s go to the answer choices:

Answer Choice (A) We have no information about governmental action. We know about news coverage of risks and the way that individuals perceive risk, but we do not have enough information to draw any conclusions about the government.

Answer Choice (B) We are only given information about the amount of coverage that two types of risk are given. We don’t know anything about the quality of risk and how that affects people. We just know that the amount that people think about risks impacts how those individuals estimate the likelihood of those potential risks coming to fruition.

Correct Answer Choice (C) We know that people estimate risk based on how often it comes to their attention. People who get information primarily from the news media (the same news media that covers rare threats more than common threats) would be likely to perceive higher risk of uncommon (i.e. rare) threats relative to common threats because of the amount of coverage they receive. This is is fully supported by our stimulus, and therefore, correct!

Answer Choice (D) We don’t get any information in our stimulus about how the element of time plays into threats or coverage of threats, so there is nothing to support this answer.

Answer Choice (E) Hard to know where to start with this one! We don't have any information about money or resources spent on threats, so we can safely (and quickly) rule this AC out.


3 comments